LSAT and Law School Admissions Forum

Get expert LSAT preparation and law school admissions advice from PowerScore Test Preparation.

 Administrator
PowerScore Staff
  • PowerScore Staff
  • Posts: 8917
  • Joined: Feb 02, 2011
|
#23953
Complete Question Explanation

Must Be True-CE. The correct answer choice is (C)

The stimulus tells us that the total spent on cataract surgery has increased over the last 10 years. The author makes the causal argument that this increase in expenditures is the result of an increase in the number of cataract surgeries, which is the result of new technology that has made cataract surgery more effective and less expensive:
  • New technology ..... :arrow: ..... More surgeries ..... :arrow: ..... Increased overall spending on surgeries
Answer choice (A): Ten years ago cataract surgery “was not always effective.” We cannot conclude from this that few had success. “Not always effective” could mean 99% effective.

Answer choice (B): This answer is far too broad and outside the scope of the stimulus.

Answer choice (C): This is the correct answer choice. If each operation has decreased in cost, and yet the total spent on the surgery has increased, then there must be more people having the operation.

Answer choice (D): While there does appear to be an increased demand for cataract surgery, this answer deals with general demand for surgery among the elderly. This topic is not discussed in the stimulus.

Answer choice (E): This is an Opposite answer. The operation was more expensive ten years ago, and likely affordable for more people at that time.
 Mastering_LSAT
  • Posts: 35
  • Joined: Jul 30, 2020
|
#92116
Hi there,

If we paraphrase the correct answer (C) using plain English, can we say: “The total amount spent on cataract surgery has increased because the increased number of people electing to have the surgery is more than to compensate for the decrease in cost per operation”?

The "offset" language is very confusing.

Many thanks!
User avatar
 atierney
PowerScore Staff
  • PowerScore Staff
  • Posts: 215
  • Joined: Jul 06, 2021
|
#92137
Hello,

Yes, this is a little confusing, mostly because I don't think the argument is particularly convincing, at least in terms of the causal relationship posited. Regardless, the idea here is that Health Care costs, as in the amount the country as a whole is spending on health care services, has gone up, and that this is due to the increased in the total of amount of health care services provided. Now, taking this as true, what also must be true or can be inferred as true on the basis of this? And the correct answer choice does this, by demonstrating that a triangle is not a circle, or we can conclude that this is a triangle because it's angles do not add up to 360 degrees. Here, if the opposite were true, in answer choice C, then the total amount spent on cataract surgery would have necessarily either diminished or stayed the same. Notice that this is the only answer choice that doesn't contain outside information or is based upon speculation, which are both disqualifications for the question type at hand.

Let me know if you have further questions on this.

Get the most out of your LSAT Prep Plus subscription.

Analyze and track your performance with our Testing and Analytics Package.